LSAT and Law School Admissions Forum

Get expert LSAT preparation and law school admissions advice from PowerScore Test Preparation.

 SherryZ
  • Posts: 124
  • Joined: Oct 06, 2013
|
#12448
Hi, amigo! Thank you so much for your continuous help!

Oct 2000 LSAT, Sec 4 LR, Q13:

AGAIN, I picked the wrong answer between two :( I chose B but the right one is D. Could you explain why D is right and B is wrong?? I appreciate it!

Have a lovely evening!

---Sherry
 SherryZ
  • Posts: 124
  • Joined: Oct 06, 2013
|
#12319
Hi, amigo! Thank you so much for your continuous help!

Oct 2000 LSAT, Sec 4 LR, Q13:

AGAIN, I picked the wrong answer between two I chose B but the right one is D. Could you explain why D is right and B is wrong?? I appreciate it!

Have a lovely evening!

---Sherry
 Steve Stein
PowerScore Staff
  • PowerScore Staff
  • Posts: 1153
  • Joined: Apr 11, 2011
|
#12328
Hola, Sherry!

Thanks for your question. That is a Flaw question, meaning that the correct answer choice must describe the flaw found in the stimulus. In this case, the author says that the soft drink is dominant, having greater than 50% of the market, but this is based on survey results showing that 72% of consumers believethat the drink dominates the market. But just because they believe it, of course, does not make it so. That is the flaw described in correct answer choice (D). Answer choice (B) describes a mistaken reversal, which is not manifest in this stimulus.

I hope that's helpful--please let me know whether this is clear--thanks!

~Steve
 Nikki Siclunov
PowerScore Staff
  • PowerScore Staff
  • Posts: 1362
  • Joined: Aug 02, 2011
|
#12449
Hello Sherry,

The survey results show that most people believe Hero to be already dominant in the market ("in the opinion of 72 percent of all consumers..."). The author takes that to mean that Hero actually dominates the market, which he defines as having more than 50% of the market share. Clearly, just because people believe Hero is dominant doesn't mean that it's actually dominant. This is a classic fallacy where the author conflates fact and opinion. Answer choice (D) highlights this flaw, and is therefore correct. There is no evidence of a MR/MN conditional error in this argument, rendering answer choice (B) incorrect.

Hope this helps!
 Jkjones3789
  • Posts: 89
  • Joined: Mar 12, 2014
|
#16598
Hello, So for this question I actually chose E and thought it was a time shift since I felt the argument was that since Harrold Food's "Hero" was at 72% and dominated the market currently it will continue to do so. Could you please explain to me why its C and the best way to go about Flaw Questions. I know all the Common ones but I still miss questions more frequently than I am comfortable with. :-? ... Thank You !
 Nicholas Bruno
PowerScore Staff
  • PowerScore Staff
  • Posts: 62
  • Joined: Sep 27, 2011
|
#16605
I think on this one, the key is to recognize that the stimulus never shows that Harold has sales in over 50% of the market. The only "fact" in the stimulus that comes close is the survey that says that 72 percent of all consumers *believe* that Harold is the dominant in the market. The fact that a consumer believes Harold is dominant does not mean the consumer buys from Harold.

Thus, C is correct because it points out that the stimulus never proves its assertion: instead it only assumes that Harold must be dominant because no other data shows the opposite.

As far as in general on Flaw questions, it is hard for me to know without seeing what you are getting wrong :). I would just say to do alot of practice tests and, on the flaw questions, focus on what seems to be "off" in the stimulus. If you read the stimulus critically you should be able to get it.

Let me know if that makes sense!
 lsatwinner68
  • Posts: 4
  • Joined: Jan 16, 2016
|
#21841
I am trying to figure out what purpose this line serves in the argument, "Since any product with more than 50 percent of sales in a market is by definition dominant in that market.."

I can see that the error is that we are using popular opinion to prove that something as being definitively true, but that line above seems to provide an actual statistical basis by what makes something dominant. If that is the case, then why can't we confidently say that Harold Foods does dominate the market based on that line, since now we have actual data to work with in addition to tibbit on popular opinion? Unless that line of course didn't directly apply to Harold's and was just a general statement then we are back to square one...
 David Boyle
PowerScore Staff
  • PowerScore Staff
  • Posts: 836
  • Joined: Jun 07, 2013
|
#21872
lsatwinner68 wrote:I am trying to figure out what purpose this line serves in the argument, "Since any product with more than 50 percent of sales in a market is by definition dominant in that market.."

I can see that the error is that we are using popular opinion to prove that something as being definitively true, but that line above seems to provide an actual statistical basis by what makes something dominant. If that is the case, then why can't we confidently say that Harold Foods does dominate the market based on that line, since now we have actual data to work with in addition to tibbit on popular opinion? Unless that line of course didn't directly apply to Harold's and was just a general statement then we are back to square one...
Hello lsatwinner68,

"Since any product with more than 50 percent of sales in a market is by definition dominant in that market.." is just a definition of dominance. For all we know, Hero is a zero, i.e., it has zero percent of the market. Maybe some misleading TV story or Wall Street Journal article got "72 percent of all consumers" to falsely believe that "'Hero' already dominates the market." So answer D is right, since the public thinking something doesn't necessarily mean that thing is true.

Hope this helps,
David
 lsatwinner68
  • Posts: 4
  • Joined: Jan 16, 2016
|
#21893
Thank you so much, I see now it was something they threw in to trick us because really it was irrelevant!
 jgray
  • Posts: 41
  • Joined: Feb 13, 2015
|
#34033
Re looking at Quest 13's stimulus I now see that they used a survey of customers to determine market share of the product.

When AC D says "Taking evidence that a claim is believed to be true to constitute evidence that the claim is in fact true" Does that mean that the author is taking the survey as the evidence?

For E, couldn't we still argue to current/future aspect as well?

thanks

Get the most out of your LSAT Prep Plus subscription.

Analyze and track your performance with our Testing and Analytics Package.